Thursday, September 21, 2023

2023/035) Prove that $4 . 29! + 5 ! \equiv 0 \pmod {31}$

We have as 31 prime  using Wilsons theorem

$30! + 1 \equiv 0 \pmod {31}$

or $30 * 29! + 1 \equiv 0 \pmod {31}$

as $30  \equiv - 1 \pmod {31}$

so we have  $-1 * 29! + 1 \equiv 0 \pmod {31}$

Or  $ 29! -1 \equiv 0 \pmod {31}$

Or $ 4 * 29! - 4 \equiv 0 \pmod {31}$

adding 124 which is multiple of 31 we get

$ 4 * 29! + 120 \equiv 0 \pmod {31}$

or $ 4 * 29! + 5! \equiv 0 \pmod {31}$

Proved 

Tuesday, September 19, 2023

2023/034) Given $a+b+c=3$ and $a^2+b^2+c^2 = 3 $ Solve for real a,b,c

We have 

$a+b+c=3\cdots(1)$

$a^2+b^2+c^2=3\cdots(2)$

Squaring (1) we get

$a^2+b^2 + c^2 + 2ab + 2bc + 2ca = 9$

Putting the value of $a^2+b^2+c^2$ from (2) we get

$ab+bc+ca = 3$

subracting above from (2) we get

$a^2+b^2+c^2 - ab - bc - ca = 0$

or $2a^2+2b^2+2c^2 - 2ab - 2bc - 2ca = 0$

or $(a^2-2ab + b^2) + (b^2-2bc+c^2) + (c^-2ca +a^2) = 0$

or $(a-b)^2 + (b-c)^2 + (c-a)^2= 0$

above is possible only if a=b= c

putting in (1) we get a = b = c = 1

2023/033) Show that if 5 does not divide n then 5$|n^4-1$

We have $n^4-1 = (n^2+1)(n^2-1)$ difference of 2 squares

 $= (n^2+ 1)(n+1)(n-1)$ difference of 2 squares

$= (n^2-4 + 5) (n+1)(n-1) as $n^2+ 1=n^2-4 + 5$ 

$= (n^2-4)(n+1)(n-1)+ 5(n+1)(n-1)$

$= (n-2)(n+2)(n+1)(n-1)+ 5(n+1)(n-1)$

$\frac{(n-2)(n-1)n(n+1)(n+2)}{n} + 5(n+1))(n-1)$

  $5(n+1))(n-1)$ is divisible by 5

$(n-2)(n-1)n(n+1)(n+2)$ being product of 5 consecutive numbers is divisible by 5

As n is not dvisible by 5 so $\frac{(n-2)(n-1)n(n+1)(n+2)}{n}$ is divisible by 5

Hence the sum is divisible by 5 and so the given expression

Note: this can be done faster using mod 5 arithmetic but I have chosen a different way.

Aditionaly this is direct from fermats little theorem 

Saturday, September 2, 2023

2023/032) Given positive integers a,b,c and $a^3+b^3 = 2^n$ prove that $a=b$

Both a nd b have to be even or add as RHS is even as it has to be > 1.

Because if a is odd and b even or vice versa then LHS is odd.

Now let us consider 1st case that is a and bare odd

So we get

$a^3+b^3 = (a+b)(a^2-ab+b^2)$

a+b is even and $a^2-ab+b^2= a(a-b) + b^2$ is odd and it has to be 1

Or $a(a-b) + b^2=1$ 

The above is possble only if a-b = 0(else  or a = b and we get a = b = 1

Now of the even case 

Let $a=p.2^q$ and b= $m.2^r$ and without loss of generality let$ q >= r$ and p and m are odd.

So we get $p^32^{3q} + m^32^{3r} = 2^n$

Devideing both sides by $2^{3r}$ we get

$p^32^{3q-3r} + m^3 = 2^{n-3r}$

RHS is even as it is minimum 2

LHS is even if 3q=3r or q = r

Puttting it we get $p^3+q^3 = 2^{n-3r}$ 

As p and q are odd so both are same and hence $a= b = 1$ for odd and $a=b=2^x$ for even  

Sunday, August 27, 2023

2023/31) Given $\frac{2+3z+4z^2}{2-3z+4z^2} \in \mathbb{R}$ and imaginary part of x is not zero find $|z|^2$

We have $\frac{2+3z+4z^2}{2-3z+4z^2}$ real 

Both numerator and denominator are expression with 3 terms 

Subtracting 1 fron the expression we shall have it real and numerator is simpler

Or   $\frac{2+3z+4z^2}{2-3z+4z^2}-1$ is real

Or$\frac{-6z}{2-3z+4z^2}$ is real

As imaglinary part of x is not zero so x is not zero so inverting 

 $\frac{2-3z+4z^2}{-6z}$ is real

Or $\frac{2-3z+4z^2}{z}$ is real

Or $\frac{2}{z}-3+4z$ is real 

And adding 3 we get   $\frac{2}{z}+4z$ is real 

Or $\frac{1}{z}+2z$ is real

Now let $z= x+ iy$

So $\frac{1}{x+iy}+2(x+iy)$ is real

Or $frac{x-iy}{x^+y^2} + 2(x+iy)|$ is real

Or $\frac{-y}{x^+y^2} + 2y)=0$

as y is non zero $\frac{1}{x^2+y^2} -2=0$ or $|z|^= \frac{1}{2}$  



Sunday, August 13, 2023

2023/030) Solve in real x $6^x + 9^x =2^(2x+1)$

As we see above power of 2 and 3 ( 9 is $3^2$) and 6 are invloved

Let $3^x = a$ and $2^x=b$

We get $ab + a^2 = 2 b^2$

Or $a^2 + ab - 2b^2 = 0$

 or $(a-b)(a+2b) = 0$

$a=b$ or $a+2b=0$

as a and b are  positive s a = b (a+2b=0 is inadmissible)

or $3^x = 2^x$ or x = 0 

Saturday, June 24, 2023

2023/029) Given $x^2+xy+ y^2=0$ find the value of $(\frac{x}{x+y})^{2023} + (\frac{y}{x+y})^{2023}$

We have $x^2+xy+ y^2= 0\cdots(1)$

 if x is zero the y is zero then x+y = 0 which is not possible as x+y is in denominator of the resultnat expression

Let $y=x\omega$

putting in (1) we get

$\omega ^2 + \omega + 1=0\cdots(2)$

Hence $\omega^3=1\cdots(2)$

Now $x+y = x + x\omega = x(1+\omega) = x(-\omega^2)= - x\omega^2$ using(1)

hence $\frac{x}{x+y} = \frac{x}{- x\omega^2} = - \omega\cdots(3)$

also  $\frac{y}{x+y} = \frac{x\omega}{- x\omega^2} = - \frac{1}{\omega} = \omega^2\cdots(4)$

hence  $(\frac{x}{x+y})^{2023} + (\frac{y}{x+y})^{2023}) = \omega^{2023} + (\omega^2)^{2023}$

= $- \omega^{2023}  - \omega^{4036}$

=  $- \omega^{3  * 677 + 1} - \omega^{3 * 1354 + 2 }$

=  $- \omega  - \omega^2 $ using (3) 

= 1 using (2)



Sunday, June 4, 2023

2023/028) what is the integer value n can take such that 3n - 10, 6n-13, and 5n-13 are prime

ew have 5n-13- (3n -10) = 2n -3 which is odd. So all 3 cannot be odd so one of them has to be 2 for all to prime.

and n has to be greater than 3 for all numbers to be positive

3n-10 = 2 gives n =4 and numebr are 2, 11, 7 all are prime 

$6n - 13 \ge 11$ and $5n - 13 'ge 7$ as $n > 3$ so other numbers cannot be 2

so only solution n = 4 

Saturday, June 3, 2023

2023/027) Find Positve integers x and m such that $\sqrt{x} + \sqrt{x+60} = \sqrt{m}$

 For the above to hold we must have $x= na^2$ and $x+60= nb^2$ where n,a,b are integers

So we get $n(b^2 - a^2) = 60$

or$n(b+a)(b-a) = 60$

now for a anb b to be integer we must have b+a and b-a to be both even or odd 

we need to find 60 as product of 3 numbers with b+ a and b- a to be different


this gives us following cases

$n=1, b + a= 30, b- a = 2$ giving $n=1,b= 16, a = 14$ giving $x= 196, m = 900$

$n=3, b + a= 10, b- a = 2$ giving $n=3,b= 6, a = 4$ giving $x= 48, m =300$

$n=4, b + a= 5, b- a = 3$ giving $n=4,b= 4, a = 1$ giving $x= 4, m = 100$

$n=4, b + a= 15, b- a = 1$ giving $n=4,b= 8, a = 7$ giving $x= 196, m = 900$ (solution repeats)

$n=5, b + a= 6, b- a = 2$ giving $n=5,b= 4, a = 2$ giving $x= 20, m = 180$

n= 15 gives a+ b and b-a to be same so no further solution

So solution set $(196,900),(48,300), (4,100),(20,180)$





Tuesday, May 30, 2023

2023/026) Show that $n!+1$ and $(n+1)!+1$ are relatively prime for all natural n ?

We shall prove it by taking the GCD

$GCD((n+1)!+1, n!+1)$

$=  GCD((n+1)!+1- (n!+1), n!+1)$ using GCD(a,b) = GCD(a-mb,b) for any integer m

$=  GCD((n+1)!- n!, n!+1)$

$=  GCD((n!(n+1-1), n!+1)$

$=  GCD(n!.n, n!+1)$

$=  GCD(n!, n!+1)$ we can devide 1st term by n and GCD shall not change and 2nd term is not divisible by n

$=  GCD( n!, n!+1-n)$  using GCD(a,b) = GCD(a-mb,b) for any integer m

$=GCD(n!,1)= 1$ 

So these are relativvly primes 


Saturday, May 20, 2023

2023/025) GIven $a+b+c = 0$ Find the value of $\frac{ab}{a^2+ab+b^2} + \frac{bc}{b^2 + bc+c^2} + \frac{ca}{c^2 + ca + b^2}$

We have $a+b+c = 0$

Hence $ a+ b = -c $

Squaring both sides $a^2+2ab + b^2 = c^2$

Adding  $a^2+b^2$ on both sides we get

$2(a^2+ab+b^2) = a^2 + b^2 + c^2$

Or $a^2+ab + b^2 = \frac{1}{2}(a^2+b^2+c^2)$

So $\frac{ab}{a^2+ab + b^2} = \frac{2ab}{a^2+b^2+c^2}\cdots(1)$

Similarly we have  $\frac{bc}{b^2+bc + c^2} = \frac{2bc}{a^2+b^2+c^2}\cdots(2)$

And $\frac{ca}{c^2+ca + a^2} = \frac{2ca}{a^2+b^2+c^2}\cdots(3)$

Adding (1) (2) and (3) we get

$\frac{ab}{a^2+ab+b^2} + \frac{bc}{b^2 + bc+c^2} + \frac{ca}{c^2 + ca + b^2}= \frac{2ab+2bc+2ca}{a^2+b^2+c^2}\cdots(4)$

Now staring with $a+b+c=0$ squaring both sides we get

$a^2+b^2+c^2 + 2ab + 2bc+2ca= 0$

Or $a^2+b^2 + c^2 = - (2ab+2bc+2ca)$

Or $\frac{2ab+2bc+2ca}{a^2 +b^2+ c^2} = -1\cdots(5)$

Form (4) and (5) we get $\frac{ab}{a^2+ab+b^2} + \frac{bc}{b^2 + bc+c^2} + \frac{ca}{c^2 + ca + b^2}= - 1$


Thursday, May 18, 2023

2023/024) show that $(x+1)^7−x^7−1$ is divisible by $x(x+1)(x^2+x+1)$

First we note that $gcd(x,x+1) = 1$

$gcd(x, x^2 + x + 1) = GCD(x, x(x+1)) = 1$

and $gcd(x+1, x^2 + x + 1) = GCD(x+1, x(x+1)) = 1$

So $x,x+1,x^2+x+1$ are pairwise co-prime and to prove that the given expression is divisible by  $x(x+1)(x^2+x+1)$  we need to show that it is divisible by $x$, $x+1$ and $x^2+x+1$

Now Let $P(x) = (x+1)^7−x^7−1$

To show that it is dvisible by x we need to show P(0) = 0

$P(0) = 1^7 - 0^7 -1 = 0$  so it is divisibe by x

To show that it is dvisible by x+1 we need to show P(-1) = 0

$P(-1) = (-1+1)^7 - (-1)7 -1 = 0$  so it is divisibe by x+1

Now we need to show that it is divisible by $x^2+x+1$

That is if $x^2+x+1=0$ then $P(x) = 0$

Because 

$x^2+x+1=0\cdots(1)$ we get

$x+1 = - x^2\cdots(2)$

And multiplying by $x-1$ we have $x^3-1 = 0$ 

Or $x^3 = 1\cdots(3)$

Now $P(x) = (x+1)^7−x^7−1$

$= (-x^2)^7 - x^7 -1$ using (1)

$=x^{14} - x^7 -1$

$= -(x^3)^4 * x^2 - (x^3)^2 * x - 1$ using (2)

$= -x^2 - x - 1$

$= - (x^2+x+1)=0$ using (1)

So P(x) =0 if $x^2+x+1=0$

Hence $x^2+x+1$ is a factor

As $x,x+1,x^2+x+1$ are factors and are pairwise co-prime so   $(x+1)^7−x^7−1$  is divisible by $x(x+1)(x^2+x+1)$ 

Wednesday, May 17, 2023

2023/023) How do you prove that $n^4−1$ is divisible by 5 if n is not a multiple of 5?

We have $n^4 -1 = (n^2+1)(n^2–1) = (n^2–4 + 5)(n^2–1) = (n^2–4)(n^2–1) + 5(n^2–1)$ 

$= (n+2)(n-2)(n+1)(n-1) + 5(n^2–1)$

now the 2nd term is multiple of 5 and 1st term is product of 4 numbers which along with are 5 consecutive numbers. so one of them has to be divisible by 5. but as n is not divisible by 5 so one of the 4 other numbers is a multiple of 5 and hence the product. as given number is sum of 2 numbers each multiple of 5 and hence the given expression.

Sunday, May 14, 2023

2023/022) Show that $\frac{n^7}{7} + \frac{n^{13}}{13} + \frac{71n}{91}$ is an integer for any integer n

 We have $\frac{n^7}{7} + \frac{n^{13}}{13} + \frac{71n}{91}$

$=\frac{n^7-n +n }{7} + \frac{n^{13}-n + n}{13} + \frac{71n}{91}$ 

$=\frac{n^7-n} {7} + \frac{n^{13}-n}{13} + \frac{n} {7}+ \frac{n}{13} + \frac{71n}{91}$ 

$=\frac{n(n^6-1)}{7} + \frac{n(n^{12}-n) }{13} + n$

If n is divisble by 7 1st term is integer and if n is not divisibl by 7 $n^6-1$ is divsible by 7 as per Fermats Little Theorem is 1st term is integer similarly the 2nd term   and 3rd term is integer for any integer n and hence the given expression  

2023/021) Find the sum of the series $1+5+10+16+23\cdots$

 Let us have a look at the $t^{th}$ term. 

Let us look at difference for a couple of terms

$t_2 = t_1 = 5- 1 = 4$

$t_3-t_2 = 10 -5 = 5$

$t_4-t_3 = 16 -10 = 6$

From the above we see that $t_k = t_{k-1} + k + 2$

From this let us find $t_n$  in terms of n

We have $t_n = 1 + \sum_{k=2}^{n} (k + 2)$

$= 1 + \frac{n(n+1)}{2} - 1 + 2(n-1)$

$= \frac{n^2}{2} + \frac{5n }{2} - 2$

So sum $S_n = \sum_{k=1}^n (\frac{k^2}{2} + \frac{5k }{2} - 2)$

$= \sum_{k=1}^n (\frac{k^2}{2}) + \sum_{k=1}^n\frac{5k }{2} - \sum_{k=1}^n2$

$=\frac{1}{2}(\frac{n(n+1)(2n+1)}{6}  + \frac{5}{2} \frac{n(n+1)}{2} - 2n $

$= \frac{2n^3 + 3n^2 +n }{12} + \frac{5n^2 +5n}{4} - 2n$

$= \frac{2n^3 + 3n^2 + n + 15n^2 + 15n -24n}{12}$

$= \frac{2n^3 + 18n^2 -8n}{12}$

$= \frac{n^3 + 9n^2 -4n}{6}$



 

 

 

Friday, May 12, 2023

2023/020) Find the minumum of $\sqrt{(x^2 + 4x + 13)} + \sqrt{(x^2 -8x + 41)}$

 We have 

 $\sqrt{(x^2 + 4x + 13)}$ Completing the square we get

$= \sqrt{x^2+ 4x+ 4 + 9} = \sqrt{(x+2)^2 + 3^2}$

The above in a plane is the distance from $(-2,-3)$ to $(x,0)$

 $\sqrt{(x^2 - 8x + 41)}$ Completing the square we get

$= \sqrt{x^2 - 8x+ 16 + 25} = \sqrt{(x-4)^2 + 5^2}$

The above in a plane is the distance from $(4,-5)$ to $(x,0)$ also from (4,5) to $(x,0)$ and because of symmetry as the line has to be to $(x,0)$ we can choose $(4,-5)$

The sum of the 2 expression is the distance from $(-2,3)$ to $(x,0)$ and from $(x,0)$ to $(4,5)$ or $(4,-5)$ as the 2 distances are same. it is better to chose a point on the other side of x axis so (4,5).

The distance ins minimum if  $(-2,3),(x,0)$ and $(4,5)$ are in same line and so minimum distance is distance from $(-2,3)$ to $(4,5)$ $= \sqrt{(4+2)^2 + (5+3)^2} = 10$

Wednesday, May 10, 2023

2023/019) Let a,b,c,d be any four real numbers but not all equal to zero. Prove that the roots of the polynomial $x^6 + a x^ 3 + bx^2 + c x + d = 0$ cannot all be real.

Because it is a degree 6 polynomial so  there are six roots let them be $x_1,x_2,x_3,x_4,x_5,x_6$ 

Because coefficient of  $x^5$  is zero so sum of roots is zero

So $\sum_{i=1}^{6}x_i = 0\cdots(1)$

Because coefficient of  $x^45$  is zero so double sum of roots is zero

So $\sum_{i=1}^{6}\sum_{j=1. j\ne i}^{6}x_i x_j = 0\cdots(2)$

Hence from (1) and (2) we have $\sum_{i=1}^{6}x_i^2 = 0$

If roots are real  all $x_i$ are zero which is impossible if a,b,c,d are non zero

Hence there is a contradiction and all roots cannot be real 



Friday, May 5, 2023

2023/018) Can the sum of three squares of odd numbers be a perfect square

An odd number is of the form 2n+ 1, so let 3 odd numbers be 2a+1, 2b+1,2c+ 1

Now $(2a+1)^2 = 4a^2 + 4a + 1 = 4(a^2+ a) + 1$

$(2b+1)^2 = 4b^2 + 4b + 1 = 4(b^2+ b) + 1$

$(2c+1)^2 = 4c^2 + 4c + 1 = 4(c^2+ c) + 1$

We observe that square of an odd is of the from 4n + 1

Now $(2a+1)^2 + (2b+1)^2 + (2c + 1)^4 = 4(a^2 + a + b^2 + b + c^2 + c) + 3$

Above is an odd number and of the form 4n + 3 so cannot be a perfect square 

So the answer is No  

Wednesday, May 3, 2023

2023/017) Simplify the following log expression

$log(10+3\sqrt{10}) + log(10+\sqrt{90+\sqrt{90}})+ log(10- \sqrt{90+\sqrt{90}})$

Solution:

Add the 2nd and 3rd expression and knowing $log\, a + log\, b = log\, ab$

  $log(10+\sqrt{90+\sqrt{90}})+ log(10- \sqrt{90+\sqrt{90}})$

  $= log((10+\sqrt{90+\sqrt{90}})(10-\sqrt{90+\sqrt{90}}))$

$= log(10^2 - (\sqrt{90 +\sqrt{90}})^2$ using $(a+b)(a-b) = a^ 2- b^2$

$=log( 100 - (90 + \sqrt{90}))$

$= log(100 - 90 - \sqrt{90})$

$= log (10 - \sqrt{3^2 * 10})$ factoring 90 to product of a square and no square

$= log( 10 - 3 \sqrt{10})$

So given expression = $log(10+3\sqrt{10})  +  log( 10 - 3 \sqrt{10})$

$= log ( (10+3\sqrt{10})(10- 3\sqrt{10}))$ knowing $log\, a + log\, b = log\, ab$

$=log (10^2 - (3\sqrt{10})$

$= log (100 - 9 * 10)$

$=log\, 10 = 1$

Monday, May 1, 2023

2023/016) Show that $6^{n+2}+7^{2n+1}$ for n positive integer is divisible by 43

 We shall prove the same by principle of mathematical induction.

Let $P(n) = 6^{n+2}+7^{2n+1}$ 

We shall show that base step and induction step are true

Base step

Here we show that P(1) is divisible by 43

$P(1) = 6^ 3 + 7^3 = 216 + 343 = 559 = 43 * 13$ is divisible by 43

So base step is true

Now for induction step

Induction step

Let it to true k that is P(k) is divisible of 43

We shall show that P(k+1) is divisible by 43

P(k) is divisible by 43 so there exists an integer b such that

$P(k) = 6^{k+2}+7^{2k+1}= 43b$ 

We have $P(k+1) = 6^{(k+1)+2}+7^{2(k+1)+1}$ 

$= 6(6^{k+1}) + 49 * 7^{2k+1} $

$= 6(6^{k+1}) + (6+43) * 7^{2k+1} $

$= 6(6^{k+1}) + 6 * 7^{2k+1} + 43 * 7^{2k+1}$

$= 6(6^{k+1} + 7^{2k+1}) + 43 * 7^{2k+1}$

$= 6 * 43 b + 43 * 7^{2k+1}$

$= 43( 6b + 7^{2k+1})$

This is multiple of 43

As we have proved both the base step and induction step hence this is true hence proved  

  

Sunday, April 23, 2023

2023/015) The sum of two numbers is 253 and their L.C.M. is 644. What are the numbers?

 Let the 2 numbers be ma and mb where GCD(a,b) = 1 and hence m is GCD of the 2 numbers and a > b

Sum = m(a+b) = 253 and LCM = mab = 644

As GCD(a,b) = 1 so gcd(a+b, ab ) =1 so m is gcd of 253 and 644

253 = 23 * 11

644 = 23 * 28

So the GCD = 23

The numbers are 23a and 23b 

$a+ b = 11\cdots(1)$

$ab = 28\cdots(2)$

So $(a-b)^2 = (a+b)^2 - 4ab = 11^2 - 4 * 28 = 9$

Or $a-b= 3\cdots(3$

Adding (1) and (3) we get $2a =14$ or $a= 7$ and putting in 1) we get b = 3  

Hence 2 numbers are 23 * 4 = 92 and 23 * 7 = 161

2023/014) Solve in real x, y $x^3+x = y^3 + y$

We have

 $x^3+x = y^3 + y$

$=>x^3-y^3 + x - y= 0$

$=>(x-y)(x^2+xy+ y^2)  + (x - y)= 0$

$=>(x-y)(x^2+xy+ y^2+1) = 0$

$=>$ $x-y = 0$ $=>x = y$

or $x^2+xy+y^2 + 1 = 0$

but $x^2+xy+y^2  +1 = (x-\frac{y}{2})^2 + \frac{3}{4}y^2 + 1 > 1$ which does not have real solution

so solution x = y 

Saturday, April 22, 2023

2023/013) How do I show that for any natural number n, the result of $1^{1987}+2^{1987}+\cdots+n^{1987} $ is not divisible by $n+ 2$

 Let us consider $2^{1987}+3^{1987}+\cdots+(n)^{1987} $

We have $k^{1987} + (n+2-k) ^{1987 }$ is divisible by n+ 2

If n is odd  taking k from 2 to $\frac{n-1}{2}$ we have |frac{n-1}{2}$ pairs and eac pair is divisible by n+2 and adding  1 does not divide by 1 

If n is even taking k from 2 to $\frac{n}{2}$ we have $\frac{n}{2}$ pairs and each pair is divisible by n+2 and middle number is $(\frac{n+2}{2})^{1987} $ 

If n+ 2 is a multiple of 4 this is even and adding 1 makes it odd and hence the sum is not divisible by n+ 2

If n+2 is of the form 4m+ 2 and in this case divisible by 2m+ 1so adding 1 does not make it divisible by 2m+ 1 so not divisible by n+ 2  proved     

Friday, April 21, 2023

2023/012) prove that there is no integer solution to $x^2 + y^2 = 3z^2$

We shall prove it by contradiction.

Let the smallest solution of the same be $(a,b,c)$ such that $a^2 + b^2 = 3c^2\cdots(1)$

Working in mod 3 we have $x^2 \equiv 0 \pmod 3\cdots(2) $ or  $x^2 \equiv 1  \pmod 3\cdots(3)$

in (1) RHS is divisible by 3 so LHS is also divisible by 3

For this to be true using (2) and (3) we must have 

$a^2 \equiv 0 \pmod 3$ 

and $b^2 \equiv 0 \pmod 3$ 

So a and b are multiples of 3 that is $a=3p$ and $b= 3q$ for some p and q

Putting in (1)

$3p^2 + 3q^2 = c^2$ or $c^2$ is multiple of 3 or c is multiple of 3

So hence $\frac{a}{3}, \frac{p}{3}, \frac{c}{3}$ is a smaller solution

Which is a contradiction

So equation does not have integer solution 

This method is known as proof by infinite descent    



Thursday, April 13, 2023

2023/011) Solve positive integers a,b,c such that abc=a+b+c

Without loss of generality assume $ a \ge b \ge c$ so $a+b+c \le 3a$

We have $abc \le 3a$ 

Or  $bc \le 3$

That is c = 1 and b =2 giving a = 3

Or c = 1 and b = 3 giving a = 2 but this violates the condition

So solution is $(3,2,1)$ or any permutation because of symmetry 

Saturday, March 18, 2023

2023/010) Given $2x = 3y-5$ find the value of $8x^3-27y^3+90xy +125$

 We have 

$2x = 3y-5$

Cubing both sides as in the required condition we have $8x^3$ and $27y^3$

We get $8x^3 = (3y-5)^3 = (3y)^3) - 5^3 -3(3y)5(3y-5)$ (using $(a -b)^3 = a^3- b^3 -3ab(a-b)$

or  $8x^3 = (3y-5)^3 = (3y)^3 - 5^3 -3(3y)*5 * 2x$ as $3y-5 = 2x$

or $8x^3 = 27y^3- 125 - 90xy$

or $8x^3-27y^3 + 90xy +125 = 0$

Thursday, March 16, 2023

2023/009) Find n such that $\lfloor \frac{n}{2} \rfloor + \lfloor \frac{n}{3} \rfloor + \lfloor \frac{n}{6} \rfloor = n$

We have $\lfloor x \rfloor \le x$ and equal ony of x is integer 

so    $\lfloor \frac{n}{2} \rfloor + \lfloor \frac{n}{3} \rfloor  + \lfloor \frac{n}{6} \rfloor \le  \frac{n}{2}  + \frac{n}{3}+ \frac{n}{6} = n$ 

so  $\lfloor \frac{n}{2} \rfloor =  \frac{n}{2}$ 

 $\lfloor \frac{n}{3} \rfloor =  \frac{n}{3}$ 

 $\lfloor \frac{n}{6} \rfloor =  \frac{n}{6}$

the above is so because if any expression above is not true LHS is less that RHS in any of them shall make the sum < n

so  $\frac{n}{2}, \frac{n}{3} ,\frac{n}{6}$ all are ntegers of n is a multiple of 2,3 and 6 that is multiple of LCM(2,3,6) that s 6. so n is of the form 6k   

Monday, March 6, 2023

2023/008) find integer n such that $(n-1)! + 1 = n^2$

We have  $(n-1)! = n^2-1 = (n+1)(n-1)$

So n-1 =0 which gives LHS = 2 RHS = 1 which is contradiction 

Or $(n-2)! = n + 1$

Put n- 2 = k giving $k! =k + 3$

As LHS is divisible by so is RHS so k is a factor of 3 k = 1 or 3

k =1 gives n = 3 which is not a solution as it does not satisfy the criteria

k =3 gives n= 5 and as $4! + 1 = 25 = 5^2$ so n = 5 is a solution 

   

Sunday, March 5, 2023

2023/007) FInd integer n such that $n^2+ 19n = n!$

 n = 0 is not a solution so $n > 0$.

deviding both sides by n we get

$n + 19 = (n-1)!$

putting n = x+ 1 we get $x+20 = x!$

for this to be valud x is a factor of 20

so we check with 1 LHS = 21 and RHS = 1

x = 2 LHS = 22 RHS = 2 no

x = 4 LHS = 24 LHS = 24 so x = 4 is a solution

x = 5 LHS = 25 RHS = 125 not a solution

if we take x larger RHS grows larger as compared to LHS so no solution

so solution x = 4 or n = 5.



Sunday, February 19, 2023

2023/006) Find 2 digit numbers that have exactly 5 factors

 If the umber is of the form $\prod_{n=1}^{k}p_n^{q_n}$ then the number of factors = $\prod_{n=1}^{k}(q_n+1)$

Because 5 is prime the number must be of the form $p^4$ where p is prime

We have the number of digits =2 and we should find p such that $ 9 \lt p^4 \lt 100$

The only number that satsfies the condition is n = 2 and n is a prime and $2^4= 16$

So 16 is the only 2 digit number having 5 factors

 



Sunday, February 5, 2023

2023/005) What is the difference of numbers if the HCF of two numbers is 9 and their LCM is 270 if the sum of the numbers is 99?

HCF is 9 so the 2 numbers are 9x, 9y where HCF(x,y) = 1 and x and y >0 and without loss of generality let x > y

product of the 2 numbers are 9xy = 270

or $xy = 30\cdots(1)$

sum of the 2 numbers are 9(x+y) = 99 or $x + y = 11\cdots(2)

we have $(x-y)^2 = (x+y)^2 - 4 * xy = 11^2 - 120 = 1$

so $x - y = 1\cdots(3)$

so we get x = 6 and y = 5 and numbers are $54, 45$ 

2023/004) Solve in integers $4^x-5^y= 39$

we have $4^x = 2^{2x}$

now working in mod 3 we get $1-5^y \equiv 0 \pmod 3$

or $5^y \equiv 1 \pmod 3$

as we know $5 \equiv -1 \pmod 3$ so y has to be even say 2m

now $4^x - 5^y = 2^{2x} - 5^{2m} = 39$

Farctoring  we get $(2^x + 5^m)(2^x-5^m) = 39 = 39 *1 = 13 * 3$ (39 can be factored in 2 ways)

so we have 2 cases

$2^x+5^m= 39$ and $2^x - 5^m=1$ adding we get $2^x *2 = 40$ and this does not have integer

or 

$2^x+5^m= 13$ and $2^x - 5^m=3$ adding we get $2^x *2 = 16$ or x = 3 and subtracting $2 * 5^m = 10$ and m = 1

so x = 3 and y = 2  

Sunday, January 29, 2023

2023/003) Prove that there exists 2023 consecutive natural numbers whose sum is a perect square

 Let the 2023 consecutive numbers be from n-1011 to n + 1011

For all to be natual numbers $n>=1012$ 

The sum of them = 2023n

$2023 =17 ^2 * 7$

So if we choose n to be of the form $7m^2$ then the sum becomes a perect square

Not $n >= 1012$ or $7m^2 >=1012$ of $m > 13$

So the 2023 number starting from $7m^2-1011$ where $m > =13$ satisfy the criteria 


Sunday, January 8, 2023

2023/002) A room is empty. Each minute a person enters a room or 2 leave. after exactily $3^{1999}$ minutes can the no of persons be $3^{1000} + 2$

Let persons enter A instances and leave B instances

A+B = $3^{1999}\cdots(1)$

A-B = $3^{1000} + 2\cdots(2)$

Subtracting 2nd from the 1st we get

$3B = 3^{1999} - 3^{1000} - 2$

LHS is multiple of 3 but RHS is not so it is not possible  

Sunday, January 1, 2023

2023/001) When dividing a polynomial f(x) by $(x-1)^2$ the remainder is x+1. If f(x) is divided by $x^2$ the remainder is 2x+3 . if the remainder when divided by $x^2(x-1)$ is $ax^2+bx+c$ then find a+b+c

 Dividing a polynomial f(x) by $(x-1)^2$ the remainder is $g(x) = x+1$-

so dividing by (x-1) the remainder is $g(1) = 1 + 1 = 2$

Dividing by $x^2(x-1) $ is $ax^2+bx+c$

so deviding  $ax^2+bx+c$ by $x-1$ remainder must be 2

so $f(1) = a + b+ c = 2$